LSAT and Law School Admissions Forum

Get expert LSAT preparation and law school admissions advice from PowerScore Test Preparation.

 Administrator
PowerScore Staff
  • PowerScore Staff
  • Posts: 8916
  • Joined: Feb 02, 2011
|
#22776
Complete Question Explanation

Must Be True-SN. The correct answer choice is (E)

This stimulus consists of the following conditional rules:

Slower to Adopt Technology → Production costs fall more slowly

Production costs fall more slowly → Unable to lower prices as rapidly

Unable to lower prices as rapidly → Squeezed out of market.

You should infer that if a country’s manufacturers are slower than are other countries’ to adopt new technology, that country will be squeezed out of the global market. The contra positive also follows: if a country is still in the global market, its companies have been adopting new technology as quickly as those of other nations.

Answer choice (A): This choice is unsupported, and incorrect. It is similar to a mistaken negation of the last statement. Furthermore, you should not assume from the stimulus that the only way to raise prices is to have squeezed foreign competitors out of the market, or even that an incapability or unwillingness to lower prices at all would lead to being squeezed out of the global market, since the stimulus was only about the relative capability to lower prices. It is possible that none of the countries could lower prices.

Answer choice (B): This is a mistaken reversal, and is incorrect.

Answer choice (C): This is a mistaken reversal, and is incorrect.

Answer choice (D): This is a mistaken negation, and is incorrect. It is necessary, not sufficient, to adopt technology.

Answer choice (E): This is the correct answer choice. The first two statements can be linked together to show that if a company does not adopt technology as quickly, it will not be able to lower its prices as rapidly. The contra positive is that if a country can lower its prices as rapidly, it adopts technology as quickly as its competitors.
 josuecarolina
  • Posts: 24
  • Joined: Jul 20, 2012
|
#4625
Okay question 30 of lesson 2 homework 2 logical reasoning questions>

#30 pretty much all the answers looked good to me on this one except A :oops:

thanks for any help/input
 Steve Stein
PowerScore Staff
  • PowerScore Staff
  • Posts: 1153
  • Joined: Apr 11, 2011
|
#4628
Hi Josue,

That's another good one. On that one, once again it helps to link everything together:

slower to adopt tech --> more slowly falling production costs --> CANNOT lower prices rapidly --> squeezed out of the market.

Then, we can take one big contrapositive, starting at the end, working backwards, and negating as we go:

NOT squeezed out --> CAN lower prices rapidly --> NOT more slowly falling costs --> NOT slower to adopt tech.

It is the diagram above that confirms answer E as the last answer choice.

I hope that's helpful! Let me know.

Thanks!

~Steve
 ellenb
  • Posts: 260
  • Joined: Oct 22, 2012
|
#8968
Dear Powerscore,

I have read the explanation for this question, however, I have a bit of trouble seeing how answer choice A and D are mistaken negations.Could you please explain?

Regards,

Ellen
 Jon Denning
PowerScore Staff
  • PowerScore Staff
  • Posts: 904
  • Joined: Apr 11, 2011
|
#8998
Hey Ellen - thanks for the question. Basically we are given a series of conditional relationships as follows:

Slower adopt tech --> prod costs fall slower --> unable lower prices as rapidly --> squeezed out of global market

Answer choice A isn't so much a mistaken negation as it is just a misstatement of the idea given in the stimulus. We know that if a country can't lower prices as rapidly as other countries then that country will get squeezed out of the global market. A says that if prices go up (so a different idea than lowering prices more slowly) it is BECAUSE they have squeezed competitors out (different than being squeezed out themselves). Emphasis on "because" up there as well, since we're dealing with conditionality in the stimulus, and A is causal (hence, wrong).

D is closer to a mistaken negation: we know if slower to adopt tech eventually that country will be squeezed out. D says adopt tech at same rate (so not slower) then neither group will be squeezed out (not squeezed out). But that deals with different relationships than the ones we're given, so it is incorrect.

I hope that helps!
 ellenb
  • Posts: 260
  • Joined: Oct 22, 2012
|
#9007
So when we negate the statement with slower we can negate it to "the same rate" or "faster". If it said faster it would still be MN correct? I just want to make sure that I get the negations correctly (when we negate a statement it is like making that part of the statement contrapositive?) (Mn for Slow-->Yellow, MN will be Not slow-->Not Yellow), but a mistaken negation can it still mean Fast-->Green (since (fast is included in the not slow category and green is included in the (not yellow) category.

Just want to make sure I have a solid grasp on MN, it seems like I got it in the lesson, but there are some examples as the above that substitute things and make me re-think what I know about MN. Just want to make sure I am clear.

Thanks

Ellen
 Jon Denning
PowerScore Staff
  • PowerScore Staff
  • Posts: 904
  • Joined: Apr 11, 2011
|
#9028
That's exactly right! Remember, negation on the LSAT is typically what we call "logical negation" (or "logical opposition"), where you essentially divide the world into two parts: the original statement, and everything else not covered by it. So the negation of "slower" is just "not slower," which could mean "faster," or "the same speed/rate."

And that's how contrapositives are formed: negate the necessary condition (remove it) and you know that you then have negated the sufficient condition (because the thing it required was just removed).

For instance, sticking with our speed/rate example, if I say "To win the race I must be the fastest" (Win --> Fastest), negating "fastest" means I won't win because that's the contrapositive, but I don't have to negate it by saying "slowest." That would certainly work, but all I know I need to negate it is "Not fastest," which might be "slowest," but it also might mean "equally fast as someone/everyone else."

A mistaken negation of that would be "I didn't win so I wasn't the fastest," and a mistaken reversal would be "I was the fastest so I won." Why are both of those possibly not true? Well maybe I was the fastest but something else kept me from winning: I was disqualified because I ran outside of my lane, or I tripped ten feet from the end and didn't finish, etc.

Very important ideas on the test so make sure you're entirely comfortable with negations, contrapositives, and conditional mistakes before test day!
 ellenb
  • Posts: 260
  • Joined: Oct 22, 2012
|
#11915
Dear Powerscore,

I am still a bit confused with answer choice A, and why it is wrong. So, it is sort of a mistaken negation, but not really?

I understand that the second part of the statement does not exactly say what we have in the stimulus it is talking about them squeezing competitors out instead of them being squueezed themeselves.

However, the first part, it does not make sense. If they raise their prices, vs. in the stimulus is if the prices will fall more slowly than their competition. (Raise their prices Not fall more slowly than their competition is going to be the CP which can include raise prices?


Thanks in advance!
Ellen
Regards,

Ellen
 BethRibet
PowerScore Staff
  • PowerScore Staff
  • Posts: 200
  • Joined: Oct 17, 2012
|
#11935
Dear Ellen,

Thanks for writing. Yes, the important factor in ruling out A is the word "because". So though you could try to interpret this answer choice as mistaken negation (loosely), as Jon has noted, the crucial factor is that the answer choice is imposing a causal relationship that isn't supported by the passage. Beyond that, I would try to be careful about not over-thinking the substance of the answer choice. As long as you notice that the cause stated here is not supported by the premises, you have more than enough information to rule out the answer choice.

Hope this helps!

Beth
 mood.iqra
  • Posts: 1
  • Joined: Jan 19, 2017
|
#32210
Dear PS,
I was successful in diagramming the problem

Slower to Adopt Technology→Production costs fall more slowly → Unable to lower prices as rapidly→ Squeezed out of market

however, i am having trouble creating the contrapositive for the question and understanding the mistaken reversals (why B and C are incorrect) and how d is a mistaken negation.

Please elaborate! Thank You!!

Get the most out of your LSAT Prep Plus subscription.

Analyze and track your performance with our Testing and Analytics Package.